User avatar
 
tamwaiman
Thanks Received: 26
Forum Guests
 
Posts: 142
Joined: April 21st, 2010
 
 
trophy
Most Thankful
 

Q18 - Marcus: For most ethical

by tamwaiman Sun Mar 06, 2011 4:47 am

I don't know why (B) is incorrect. Anita says that Marcus's point is inadequate, does it mean that Anita says Marcus wrong?

Thank you.
User avatar
 
ManhattanPrepLSAT1
Thanks Received: 1909
Atticus Finch
Atticus Finch
 
Posts: 2851
Joined: October 07th, 2009
 
This post thanked 1 time.
 
 

Re: Q18 - Marcus: For most ethical

by ManhattanPrepLSAT1 Mon Mar 07, 2011 4:45 pm

Marcus makes three points - one of which Anita disagrees with. Marcus claims that traditional journalistic ethics is clear, adequate, and essentially correct. Anita disagrees with the claim that it's adequate, but NOT with with the claim that it's correct.

Answer choice (B) is about whether traditional journalistic ethics are correct. This is not something that Anita is addressing.

Does that answer your question here on answer choice (B)?
 
nonameee
Thanks Received: 0
Forum Guests
 
Posts: 10
Joined: December 19th, 2011
 
 
 

Re: Q18 - Marcus: For most ethical dilemmas the journalist is li

by nonameee Mon Dec 19, 2011 8:58 am

I want to make sure that I correctly understand why option (D) is wrong:

(D) is wrong because it states a very broad and unsubstantiated generalization that there are situation in which NO principle of journalistic ethics can be of any help.

Am I right?

Thank you.
User avatar
 
ManhattanPrepLSAT1
Thanks Received: 1909
Atticus Finch
Atticus Finch
 
Posts: 2851
Joined: October 07th, 2009
 
 
 

Re: Q18 - Marcus: For most ethical dilemmas the journalist is li

by ManhattanPrepLSAT1 Fri Dec 23, 2011 6:02 pm

nonameee Wrote:I want to make sure that I correctly understand why option (D) is wrong:

(D) is wrong because it states a very broad and unsubstantiated generalization that there are situation in which NO principle of journalistic ethics can be of any help.

Am I right?

That's exactly right. Answer choice (D) is much broader than what Anita is suggesting. She merely states that the principle outlined by Marcus would be of no help, not that "no" principle would be of any guidance.

Let's just walk through this one from the beginning. We're asked to find Anita's conclusion - which is that "[in some cases] this guidance is inadequate." Her comment is reflexive and refers back to the guidance outlined by Marcus in his statements. So we simply need an answer choice that says that the principle outlined by Marcus will not always provide proper guidance - best expressed in answer choice (C).

Let's look at the incorrect answer choices:

(A) would suggest that Anita undermines a different claim made by Marcus than she actually challenges. Anita does not address most ethical dilemmas, but rather a typical case when a decision needs to be made.
(B) would suggest that Anita undermines a different claim made by Marcus than she actually challenges. Anita does not address most ethical dilemmas, but rather a typical case when a decision needs to be made.
(D) is too strong. It's not the case that "no" principle can offer guidance, but simply that Marcus' principle would not.
(E) is too extreme. Anita says that the principle does not offer guidance in every situation, but does not say that traditional journalistic ethics are no more than an unnecessarily convoluted description of the journalist's job.

Nice work nonameee!
 
nonameee
Thanks Received: 0
Forum Guests
 
Posts: 10
Joined: December 19th, 2011
 
 
 

Re: Q18 - Marcus: For most ethical dilemmas the journalist is li

by nonameee Sat Dec 24, 2011 5:57 am

mshermn, thanks a lot for your confirmation!
 
aznriceboi17
Thanks Received: 5
Elle Woods
Elle Woods
 
Posts: 76
Joined: August 05th, 2013
 
 
trophy
Most Thankful
 

Re: Q18 - Marcus: For most ethical

by aznriceboi17 Mon Jan 20, 2014 3:54 am

I'm a bit thrown off by the use of the word 'typical'. When we refer to 'typical' cases from some set, does that not actually refer to 'most' cases from that set?
User avatar
 
ManhattanPrepLSAT1
Thanks Received: 1909
Atticus Finch
Atticus Finch
 
Posts: 2851
Joined: October 07th, 2009
 
This post thanked 1 time.
 
 

Re: Q18 - Marcus: For most ethical

by ManhattanPrepLSAT1 Sun Jan 26, 2014 2:41 pm

You're absolutely correct aznriceboi17.

My reference to "some" is regarding ethical dilemmas. Because the "typical case" described by Anita is regarding decisions about whether the information is newsworthy, we can say that for such cases, the guidance outlined by Marcus is typically (most of the time) inadequate.

If we expand the subject to ethical dilemmas more generally, as Marcus discusses, we cannot say that the guidance is typically inadequate. Remember Anita only addresses a subset of ethical dilemmas--those regarding the determination of whether information is newsworthy.

Hope that helps!
 
jj3.romero
Thanks Received: 0
Vinny Gambini
Vinny Gambini
 
Posts: 1
Joined: September 01st, 2016
 
 
 

Re: Q18 - Marcus: For most ethical

by jj3.romero Thu Sep 01, 2016 5:43 pm

Hi,

I chose answer C for this question, but I'm still wondering why A is not also correct. For A to be correct, all that must be shown, as I understand it, is that traditional journalistic ethics is clear for not-most ethical dilemmas, thereby showing that Marcus' claim is incorrect.

However, if we use the definition of "typical" above where it indicates "most," which I think is a valid inference, it seems that if C is correct then A is also correct. For, if "the ethical principle that Marcus cites does not help the journalist in a typical kind of situation" (C) wouldn't that also mean that the ethical principle is not adequate for most ethical dilemmas, thereby proving that A is correct that the "claim that traditional journalistic ethics is clear for most ethical dilemmas... is incorrect"? Even if "typical" doesn't connote or denote "most," it surely connotes a significant proportion of all situations in which a decision needs to be made, and thus would mean that traditional journalistic ethics is clear for not-most ethical dilemmas, meaning that A would be correct.

Where am I going wrong here? Thanks for any insight you can share!

Sincerely,

jj3